Calculating f(x) PDF Mean,Median,Mode & Plotting

  • Thread starter Thread starter abhipatel
  • Start date Start date
  • Tags Tags
    Pdf
Click For Summary
The discussion focuses on determining constants A and B for the function f(x) = Ax - Bx^3 to ensure it is a valid probability density function (PDF) with a mean of 0.9 times its mode. Participants discuss the necessary equations derived from setting the mean equal to 0.9 times the mode and ensuring the integral of the PDF equals 1 over its domain. The calculated values are A = 0.8466, B = 0.1733, Mean = 1.1485, Median = 1.1724, and Mode = 1.2761. There is also clarification on the definitions of mean and mode in the context of PDFs and the integration process involved. Overall, the conversation helps clarify the steps needed to derive the solution.
abhipatel
Messages
26
Reaction score
0
Determine constants A and B such that function f(x) is a valid PDF whose mean is 0.9 times its mode.

f(x) = Ax- Bx ^3 , 0 <= x <= 2, otherwise = 0

Also calculate its mean,median and mode and plot the PDF

Please help...am scratching my head...have the answers but cannot derive the solution

Answers : A = 0.8466 ; B = 0.1733 ; Mean = 1.1485 , Median = 1.1724 , Mode = 1.2761
 
Physics news on Phys.org
What are the definitions of mean and mode for a PDF? Set mean=mode*0.9. That gives you one equation in A and B. Now remember a PDF needs to have integral=1 over it's domain. That gives you another equation for A and B. Solve them.
 
got one function...believe am screwing up in equating the mode and the mean i believe...mean is integral of x.f(x)dx right? basically integrate the function with limits 0 & 2...allright so u get 8A/3 - 32B/5 as Mean...Mode...am doing df(x)/dx...i don't know if that is correct?? thanks though am getting a good picture now...just a lil twist needed..
 
got it my friend...excellent help..thanks a lot buddy..take care
 
Question: A clock's minute hand has length 4 and its hour hand has length 3. What is the distance between the tips at the moment when it is increasing most rapidly?(Putnam Exam Question) Answer: Making assumption that both the hands moves at constant angular velocities, the answer is ## \sqrt{7} .## But don't you think this assumption is somewhat doubtful and wrong?

Similar threads

Replies
3
Views
1K
  • · Replies 8 ·
Replies
8
Views
2K
  • · Replies 4 ·
Replies
4
Views
2K
  • · Replies 3 ·
Replies
3
Views
3K
  • · Replies 3 ·
Replies
3
Views
1K
Replies
4
Views
10K
  • · Replies 1 ·
Replies
1
Views
3K
  • · Replies 6 ·
Replies
6
Views
3K
Replies
3
Views
2K
  • · Replies 15 ·
Replies
15
Views
2K